Example) Mathematical Statistics and Data
Analysis 3RD Edition Chapter 8, Q4
Suppose that X is discrete random variable
with P(X=0)=$\frac{2}{3}\theta$, P(X=1)=$\frac{1}{3}\theta$,
P(X=2)=$\frac{2}{3}(1-\theta)$, P(X=3)=$\frac{1}{3}(1-\theta)$. The following
10 independent observations were taken from such a distribution. (3, 0, 2, 1,
3, 2, 1, 0, 2, 1)
a) Find the method of moment estimate of
$\theta$.
b) Find the approximate standard error for
your estimate.
▷Think first!
- Based on the sample observations with
size 10, what is the sample mean?
- How can you find the expected value in
discrete case?
- From the results from two questions
above, what is your conclusion?
- We can find the sample variance by using
the second moment. So what is variance of the estimate?
▷Solution (a)
Sample mean
$=\frac{3+0+2+1+3+2+1+0+2+1}{10}$=$\frac{15}{10}$=$\frac{3}{2}$
$E[X^2]=0\cdot P(X=0) + 1\cdot
P(X=1)+2\cdot P(X=2) + 3\cdot P(X=3)$
= $0\cdot \frac{2}{3}\theta + 1\cdot \frac{1}{3}\theta+2\cdot
\frac{2}{3}(1-\theta)+3\cdot \frac{1}{3}(1-\theta)$
= $-2\theta + \frac{7}{3}$
$\therefore$ $\frac{3}{2} = -2\theta +
\frac{7}{3} \rightarrow \theta = \frac{5}{12} \rightarrow
\hat{\theta}=\frac{5}{12}$
▷Solution (b)
The variance of estimate,
$Var(\hat{\theta}) = Var (\frac{7}{6}-\frac{1}{2}\bar{X})$ =$({\frac{1}{2}})^2
+ Var(\bar{X}) = \frac{1}{4}\cdot \frac{\sigma^2}{n}$ = hummm... what is the
variance of X?
Variance = $\sigma ^2=E[X^2] - [E(X)]^2$,
so we need the second moment!
$E[X^2]=0^2\cdot P(X=0) + 1^2\cdot
P(X=1)+2^2\cdot P(X=2) + 3^2\cdot P(X=3)$
$=\theta (\frac{1}{3}-\frac{8}{3}-\frac{9}{3})+ \frac{8}{3}+\frac{9}{3}=
-\frac{16}{3}\theta +\frac{17}{3}$
$\sigma ^2=E[X^2] - [E(X)]^2 =
-\frac{16}{3}\theta +\frac{17}{3} -(-2\theta+\frac{7}{3})^2 = -2\theta
^2+4\theta+\frac{2}{9}$
Let's go back the variance of estimate,
$Var(\hat{\theta})=\frac{1}{4}\cdot \frac{\sigma^2}{n}=\frac{1}{40}(-4\theta
^2+4\theta +\frac{2}{9})$
Now substituting $\theta$ by its moment
estimate $\hat{\theta}$, the estimated variance is
$Var(\hat{\theta})=\frac{1}{40}(-4\hat{\theta}^2+4\hat{}\theta
+\frac{2}{9})$ = <$\hat{\theta}=\frac{5}{12}$
from (a)>
$=\frac{1}{40}(-4\theta ^2+4\theta +\frac{2}{9})=\frac{1}{40}\cdot
\frac{172}{144}=0.029861$
$\therefore$
$SE(\hat{\theta})=\sqrt{Var(\hat{\theta})}=\sqrt{0.029861}=0.1728$
No comments:
Post a Comment